Ln x 1 0: Решите уравнение ln(x+1)=0 (ln(х плюс 1) равно 0)

3-8 9 Оценить квадратный корень из 12 10 Оценить квадратный корень из 20 11 Оценить квадратный корень из 50 94 18 Оценить квадратный корень из 45 19 Оценить квадратный корень из 32 20 Оценить квадратный корень из 18 92

исчисление — Как доказать, что $\log(x)1$?

спросил

Изменено 11 месяцев назад

Просмотрено 21к раз

$\begingroup$

Это очень просто, но мне трудно найти способ доказать это неравенство

$\log(x)

когда $x>1$

($\log(x)$ — натуральный логарифм)

Я могу думать о двух графиках, но не могу найти другой способ доказать это, и, кроме того, я не понимаю, почему это не должно выполняться, если $x<1$

Кто-нибудь может мне помочь?

Заранее спасибо.

  • исчисление
  • неравенство
  • логарифмы

$\endgroup$

5

$\begingroup$

Вы можете просто различать $$ f(x):=\log x-x, \quad x\geq1, $$ давая $$ f'(x)=\frac1x-1=\frac{1-x}x<0 \quad \text{for}\quad x>1 $$ с $$ f(1)=-1<0 $$ и $f$ строго убывает, то $$ f(x)<0, \quad x>1, $$ это $$ \log x -x <0, \quad x>1. $$

$\endgroup$

2

$\begingroup$

909х\ле \фрак{1}{1-х}$$

Логарифмирование обеих частей $(4)$ дает желаемое неравенство

$$\log(1+x)\le x \tag 5$$

+1)$ в $(4)$ показывает

$$\log(1+z)\ge \frac{z}{z+1}$$

для $z>-1$. Собрав все вместе, мы имеем для $x>0$

$$\frac{x-1}{x}\le \log x\le x-1

$\endgroup$

6 9{1}{\frac{1}{t}\,dt}\le 0 < x.

$$

$\endgroup$

$\begingroup$

Я предполагаю, что вы знаете производную от $\log$.

Пусть $f(x)=\log x -x$. Затем $$f'(x) = \frac 1x -1<0\ \\forall x>1.$$ Более того, $f(1) = -1<0$. Итак, у вас есть функция, которая начинается с отрицательного значения при $x=1$, а затем уменьшается, поскольку ее производная всегда отрицательна. Это значит, что $$f(x) = \log(x) - x <0\ \\для всех x>1,$$ что вы хотели показать. 9x) > \log(x)$, так как $\log$ увеличивается. Следовательно, $x > \log(x)$ для $x\geq0$.

$\endgroup$

0

$\begingroup$

У вас даже есть $\;\log x \le x-1$, потому что $\log$ является вогнутой

функцией, а линия с уравнением $y=x-1$ является касательной к графику $\log$ в $(1,0)$. Следовательно: $$\log x \le x-1

$\endgroup$

$\begingroup$

Определите $f(x) = \log x — x$. Теперь $f'(x) = \frac{1}{x}-1$, что отрицательно, если $x > 1$. Таким образом, $f$ строго убывает на интервале $(1, \infty)$.

Теперь, поскольку $f(1) = \log 1 — 1 = 0-1 = -1$, мы должны иметь $f(x) < -1$ на $(1, \infty)$. Таким образом, $\log x - x < -1 < 0$ на $(1, \infty)$. Отсюда следует, что $\log x < x$ при $x > 1$.

$\endgroup$

0

$\begingroup$

Когда $x=1$, $\log x=0<1=x$. Далее, для $x>1$ имеем $\frac{d}{dx}\log x=\frac{1}{x}<1=\frac{d}{dx}x$.

Это показывает, что $x$ больше $\log x$ при $x=1$ и что $x$ растет быстрее, чем $\log x$ при $x>1$. Следовательно, $x>\log x$ для $x\ge 1$.

$\endgroup$

$\begingroup$

Различные способы выполнения этого упражнения, безусловно, зависят от того, что вы хотите предположить. + \to \mathbb{R}$, удовлетворяющим $$ f(xy) = f(x)+f(y) \tag{1} $$ это немедленно дает $f(1)=0, f(x)+f(\frac1{x})=0$, поэтому $f$ является нетривиальным гомоморфизмом абелевой группы с $\exists c\dot f(c) \ne 0$ 9+$ (1) означает, что $f$ сохраняет порядок или инвертирует порядок в зависимости от знака $\log c$ и от того, $c \gt 1$. таким образом, чтобы исключить порядковые антиизоморфизмы, нам потребуется еще одно предположение, что $f((1,\infty)) \subseteq (0,\infty)$

предположим, что $f$ имеет неподвижную точку $\zeta \gt 1$. то есть точка, для которой в качестве действительных чисел $$ f(\дзета) = \дзета $$ мы покажем, что это приводит к противоречию.

поскольку $f(1)=0$ и $f$ строго монотонна и непрерывна, уравнение $f(x)=1$ имеет единственное решение, скажем, $x=e \gt 1$. 92}$, который всегда отрицателен. Это означает, что любая касательная к графику $y=\ln(x)$ будет больше или равна $\ln(x)$, причем равенство достигается только в точке касания. Тогда мы можем заключить, что касательная $x-1$ больше или равна $ln(x)$.

Добавить комментарий

Ваш адрес email не будет опубликован. Обязательные поля помечены *